4
$\begingroup$

Let $X$ be a smooth, connected, complex analytic variety, and $Y\subset X$ a closed, analytic subvariety of codimension at least 2. Now let $V\subset X\backslash Y$ be a closed, analytic subvariety. Is the closure $\bar{V}$ of $V$ in $X$ also analytic?

Motivation: In the case where $X$ is a surface, this seems to follow from Riemanns extension theorem, and perhaps it can even be proven this way. But it'd be nice if there were a reference (or if I'm missing something and this is false in general!)

Thanks!

$\endgroup$

1 Answer 1

5
$\begingroup$

Yes, by Remmert-Stein's extension theorem. See e.g. Fritsche-Grauert, From holomorphic functions to complex manifolds, Theorem 6.9.

$\endgroup$
5
  • $\begingroup$ Perfect, thanks! This theorem (rightly) says that I need to assume that $V$ is of dimension larger than $Y$, otherwise i could take for example $X=\mathbb{P}^1, Y=\{0\}, V=\cup_n \{\frac1n\}$. If I assume $V$ is connected, can I remove the restrictions on dimension? $\endgroup$
    – jacob
    Feb 10, 2018 at 22:33
  • $\begingroup$ You could take $X=\mathbb{P}^1\times\mathbb{C}$, $Y=\{0\}\times \mathbb{C}$, and $V=((\cup_n \{\frac{1}{n}\})\times\mathbb{C})\cup ((\mathbb{P}^1\setminus\{0\})\times\{1\})$. $V$ is connected. $\endgroup$
    – Qfwfq
    Feb 10, 2018 at 22:59
  • $\begingroup$ Or, if you want $V$ irreducible, take $X=\mathbb{C}^2$, $Y=\{0\}\times\mathbb{C}$, and $V=\{ (z,\exp(\frac{1}{z}))\in \mathbb{C}^* \times \mathbb{C} \mid z\in\mathbb{C}^* \}$. $\endgroup$
    – Qfwfq
    Feb 10, 2018 at 23:06
  • $\begingroup$ Yeah I meant irreducible. In the second case, $Y$ doesn't satisfy my initial assumption of codimension at least 2 (this was sort of the motivation). Of course thats sort of a moot point cause you could just embed the whole thing in a bigger space eh. Ok, thanks for your help. I'll consolidate my thoughts and maybe post more later. $\endgroup$
    – jacob
    Feb 10, 2018 at 23:16
  • $\begingroup$ Oh yes, you can cross everything by $\mathbb{C}$ and obtain higher codim, as you say. $\endgroup$
    – Qfwfq
    Feb 10, 2018 at 23:18

Your Answer

By clicking “Post Your Answer”, you agree to our terms of service and acknowledge that you have read and understand our privacy policy and code of conduct.

Not the answer you're looking for? Browse other questions tagged or ask your own question.